aka a way to use algebra to mask the fact that actual combo is hard
I haven't done a true math post in a while, so here goes....
Chapter 1 Key Idea: Let

be a finite graph on

vertices (not necessarily simple), and let

denote its adjacency matrix. Then the number of closed walks on

of length

is
![\[f_G(\ell)=\sum_{i=1}^n(A(G)^\ell)_{i,i}=\operatorname{tr}\left(A(G)^\ell\right)=\lambda_1^\ell+\lambda_2^\ell+\cdots+\lambda_n^\ell,\]](//latex.artofproblemsolving.com/6/0/9/609bcc4e4fdcc62bdca9c7de8ce776ad99658610.png)
where

is the sequence of eigenvalues of
. (Note that all the

are real by the Spectral Theorem.)
This isn't
too hard to prove (and is probably made easier based on the wording of the statement). Now on to the problems I guess?
Stanley Chapter 1 Exercise 2 wrote:
Suppose that the graph

has

vertices and that the number of closed walks of length

in

is
![\[8^\ell+2\cdot 3^\ell+3\cdot(-1)^\ell+(-6)^\ell+5\]](//latex.artofproblemsolving.com/8/7/b/87be345b574f9b30a4b84d9aa63d27843f6c0fe4.png)
for all
. Let

be the graph obtained from

by adding a loop at each vertex (in addition to whatever loops are already there). How many closed walks of length

are there in
?
SolutionTo kill this problem, we need a somewhat useful lemma.
LEMMA: Suppose
,
, 
and
,
, 
are nonzero complex numbers such that for all positive integers
, we have
![\[\alpha_1^\ell+\cdots+\alpha_r^\ell=\beta_1^\ell+\cdots+\beta_s^\ell.\]](//latex.artofproblemsolving.com/4/c/3/4c33b8a747861a5d3e9a6bc347ce5d1f3c5cbe05.png)
Then

and the
's are just a permutation of the
's.
Proof. The following proof is due to Richard Stanley (the author of the textbook I'm working from). We use generating functions. Let

be a complex number whose modulus is close to
. Multiply the displayed equation by

and sum on all
. The geometric series we obtain will converge, and we get
![\[\dfrac{\alpha_1x}{1-\alpha_1x}+\cdots+\dfrac{\alpha_rx}{1-\alpha_rx}=\dfrac{\beta_1x}{1-\beta_1x}+\cdots+\dfrac{\beta_sx}{1-\beta_sx}.\]](//latex.artofproblemsolving.com/f/3/e/f3e4360f0965b0fe5786b24b9d40ed62160fdf3a.png)
This is an identity valid for complex numbers of sufficiently small modulus. By clearing denominators we obtain a polynomial identity. But if two polynomials in

agree for infinitely many values, then they are the same polynomial, so the above is actually valid for all

(excluding ones which give rise to a zero denominator).
Fix a complex number
. Multiply the above by

and let
. The left hand side becomes the number of
's which are equal to
, while the right hand side becomes the number of
's which are equal to
. Hence these numbers agree for all
, so the lemma is proved.
From here, the problem is simple. Remark that since

is obtained by adding a loop to each vertex of
, the adjacency matrix of

is the same as the adjacency matrix of

but with

added to each diagonal entry. Thus
, meaning that the eigenvalues of

are one greater than the eigenvalues of
. (Note: the eigenvalues of a graph are just the eigenvalues of its adjacency matrix.) From the lemma, the given condition on
, and the fact that

has

vertices, we see that the eigenvalues of

are
,
,
,
,
,
,
,
,
,
,
,
,
,
,
. (Do you see why?) Hence the eigenvalues of

are
,
,
,
,
,
,
,
,
,
,
,
,
,
,
, meaning that the number of closed walks of length

in

is
![\[\boxed{9+2\cdot 4^\ell+5\cdot 2^\ell+(-5)^\ell+3}.\]](//latex.artofproblemsolving.com/b/b/8/bb8fdea23059941b3e6066b6126419483f063c2c.png)
Stanley Chapter 1 Exercise 3 wrote:
A bipartite graph

with vertex bipartition

is a graph whose vertex set is the disjoint union

of

and

such that every edge of

is incident to one vertex in

and one vertex in
. Show that the nonzero eigenvalues of

come in pairs
. Equivalently, prove that the characteristic polynomial of

has the form

if

has an even number of vertices or

if

has an odd number of vertices for some polynomial
.
SolutionNote that since

is bipartite, a walk must consist of the sequence of vertices
,
,
,
,
,
, where

and

for all

and
. As a result, no closed walk in

can be of even length. This means that if
,
, 
are the eigenvalues of
, then
![\[\lambda_1^\ell+\cdots+\lambda_n^\ell=0\qquad\text{for all odd }\ell.\]](//latex.artofproblemsolving.com/c/2/c/c2c5452fd125900aa0a84fc2f443fbb4a69c845f.png)
Now for each positive integer
, let

denote the sum of the

powers of the eigenvalues and

denote their

symmetric sum. We now prove a lemma.
LEMMA: 
for all odd
.
Proof. We use strong induction and Newton's Sums. *looks up what Newton's sums are again* The base case,
, is trivial. Now assume

for all
, where by assumption

is odd. Recall that
![\[P_{i+2}-S_1P_{i+1}+S_2P_i-\cdots+S_{i+1}P_1-(i+2)S_{i+2}=0.\]](//latex.artofproblemsolving.com/2/9/f/29f42f9817ef9878fd502ca24ef93d409327778b.png)
Consider the expression

for some arbitrary
. Remark that since

is odd, exactly one of

or

must be odd. If

is odd, then

by assumption, while otherwise

from the original problem statement. Thus

for each
, and so the entire equality reduces to
![\[-(i+2)S_{i+2}=0\quad\implies\quad S_{i+2}=0.\]](//latex.artofproblemsolving.com/c/8/9/c897dafac635345bc800083a4af190a6db8692ad.png)
Hence we're done by induction.
This allows us to finish off the problem easily. Let
![\[\lambda(A(G))=\sum_{i=0}^na_i\lambda^i\]](//latex.artofproblemsolving.com/a/a/7/aa7230c08ed6dfc75a56213023056da7587fad3c.png)
be the characteristic polynomial of

for some sequence of coefficients
. Then by Vieta's Formula's we know
, and so it is not hard to see that
![\[\lambda(A(G))=\begin{cases}a_n\lambda^n+a_{n-2}\lambda^{n-2}+\cdots+a_0,&n\text{ even},\\a_n\lambda^n+a_{n-2}\lambda^{n-2}+\cdots+a_1\lambda,&n\text{ odd}\end{cases}\]](//latex.artofproblemsolving.com/6/7/f/67f4a48f4001e94104b68a1b896bfe774ac6399a.png)
as desired.
Stanley Chapter 1 Exercise 5 wrote:
Let

be the complete bipartite graph

with

vertex-disjoint edges removed. Thus

has

vertices and

edges, each of degree
. Show that the eigenvalues of

are
(
times each) and

(once each).
SolutionWe first seek to gather information about the adjacency matrix
. Denote by

the vertex bipartition of
. WLOG, number the vertices in

labels

through

and the vertices in

labels

through

such that

(or rather the edge set of
) for each
. (The basic idea behind this is to introduce some consistency in the labeling. The advantages of such a system will become apparent later.) Note that

for all

and

by our labeling system. Furthermore, note that with respect to the remaining entries of
, we will only get a nonzero entry in the

and

entries. Thus, we can conclude that our adjacency matrix has the form
![\[A(H_n)=\begin{bmatrix}0&J-I\\J-I&0\end{bmatrix},\]](//latex.artofproblemsolving.com/e/2/a/e2a26a1cbe4424be2f66c73e6ad8c1b506f624e7.png)
where

is the matrix with all ones. For example, when
, we get
![\[A(H_3)=\begin{bmatrix}0&0&0&0&1&1\\0&0&0&1&0&1\\0&0&0&1&1&0\\0&1&1&0&0&0\\1&0&1&0&0&0\\1&1&0&0&0&0\end{bmatrix}.\]](//latex.artofproblemsolving.com/8/1/4/814a7f4fdebf592fbb01cc9938a1a22b9c424f10.png)
To compute the eigenvalues of this matrix, recall that if
,
,
, and

are square blocks, then
![\[\det\begin{bmatrix}A&B\\C&D\end{bmatrix}=\det(AD-BC).\]](//latex.artofproblemsolving.com/b/8/d/b8d4d9c6c2a3330125112ce4015dc1941498967d.png)
(I seriously need to find the proof for this....) Now for any
, note that
![\[\det[A(H_n)-\lambda I]=\det\begin{bmatrix}-\lambda I&J-I\\J-I&-\lambda I\end{bmatrix}=\det(\lambda^2 I-(J-I)^2).\]](//latex.artofproblemsolving.com/b/3/7/b370ae17dcf086beed2fcc66a08be1775733f5df.png)
Using the fact that

and

commute and that

(why?), remark that
![\[(J-I)^2=J^2-2IJ+I^2=nJ-2J+I=(n-2)J+I.\]](//latex.artofproblemsolving.com/c/1/6/c1617138d9534169849f87c324b1c04db275f5b7.png)
Thus
![\[\det[\lambda^2I^2-(I-J)^2]=0\iff \det[J(n-2)-(\lambda^2-1)I]=(n-2)\det\left[J-\left(\dfrac{\lambda^2-1}{n-2}\right)I\right]=0.\]](//latex.artofproblemsolving.com/5/f/c/5fc0303606b034131782d5bef51af3b11ccc1cb4.png)
(This obviously cannot be done for
, but this is an easy special case to deal with.)
Now we use a lemma. (The reason I'm putting this part off as a lemma is because its proof was mentioned within the chapter.)
LEMMA: If

is
, then the eigenvalues of

are

with multiplicity

and

with multiplicity
.
Proof. Note that
, so

of the eigenvalues of

must be zero (proof: row reductions). To compute the last eigenvalue, note that the trace of

is
. Since the trace of a matrix is the sum of the eigenvalues of that matrix, we see that the last eigenvalue is

as desired.
Now let
. Then
, so

is an eigenvalue of
. By the lemma, this means that

with multiplicity

or

with multiplicity
. Now plugging back in, we see that either
![\[0=\dfrac{\lambda^2-1}{n-2}\quad\implies\quad \lambda=\pm 1\]](//latex.artofproblemsolving.com/f/d/4/fd4cb25bd43145c02075662040c0c6534a42c021.png)
or
![\[n=\dfrac{\lambda^2-1}{n-2}\quad\implies\quad \lambda=\pm (n-1)\]](//latex.artofproblemsolving.com/8/b/6/8b6fc12824058a8daec4b1f152759838178c2454.png)
and it's not hard to show that these roots have the desired multiplicities, so we're done.
Stanley Chapter 1 Problem 11 wrote:
Let

denote the complete graph with

vertices, with one loop at each vertex. Let

denote

with the edges of

removed, i.e. choose

vertices of

and remove all edges between these vertices (including loops). Find the number

of closed walks in

of length
.
SolutionOnce again, it first suffices to number the vertices of

in an advantageous manner and then consult its adjacency matrix. WLOG number the vertices

such that the three vertices
not chosen in

are
. Then the vertices labeled

are connected to every vertex in
, and furthermore these are the only edges in

since all others were removed. Hence, the adjacency matrix is
![\[A(\Gamma)=\begin{bmatrix}1&1&1&1&\cdots&1\\1&1&1&1&\cdots&1\\1&1&1&1&\cdots&1\\1&1&1&0&\cdots&0\\\vdots&\vdots&\vdots&\vdots&\ddots&\vdots\\1&1&1&0&\cdots&0\end{bmatrix}.\]](//latex.artofproblemsolving.com/5/d/6/5d612a821e14544e1b46b9e00e6c6b80ad1e61a5.png)
In other words,

consists of two bands of
s, each of length
, and zeroes everywhere else.
In order to compute the number of closed walks of arbitrary length, it suffices to find the eigenvalues of
. It is most likely possible to do this by actually determining the characteristic polynomial, but this is quite arduous and does not give rise to any meaning behind the problem; instead, we choose an alternate route. Remark that by inspection
, so all but two of the eigenvalues are nonzero. To compute the values of the nonzero eigenvalues, we look to trace. Recall that the eigenvalues of a square matrix

are the squares of the eigenvalues of the original matrix
. With this in mind, we attempt to determine the diagonal entries of
. This is actually not so hard to do! Note that by the definition of matrix multiplication
![\[(A(\Gamma)^2)_{ii}=\sum_jA(\Gamma)_{ij}A(\Gamma)_{ji}.\]](//latex.artofproblemsolving.com/4/a/b/4aba38c4fceac3aa8ecdc88459315541be9f3993.png)
If
, then all entries of the form

or

are equal to
, and so the corresponding entry equals
. Otherwise, only the three nonzero entries in the corresponding row/column count toward the final sum, and so the corresponding entry equals
. Hence
![\[A(\Gamma)^2=\begin{bmatrix}21&*&*&*&\cdots&*\\ *&21&*&*&\cdots&*\\ *&*&21&*&\cdots&*\\ *&*&*&3&\cdots&*\\\vdots&\vdots&\vdots&\vdots&\ddots&\vdots\\ *&*&*&*&\cdots&3\end{bmatrix},\]](//latex.artofproblemsolving.com/c/6/d/c6dc774563d73ce0efdfcb3dcf8584d8c072c859.png)
meaning that
Finally, let

and

be the two nonzero eigenvalues of
. Since

trivially, we get the system of equations
![\[\begin{cases}\lambda_1+\lambda_2&=3,\\\lambda_1^2+\lambda_2^2&=117.\end{cases}\]](//latex.artofproblemsolving.com/f/0/8/f08a1408ad8ac1d4a9360467081500d5c0f343cb.png)
Solving yields

and

as the solutions to this system. As a result, from the main idea the number of closed walks of length

is
.
Solution(a)Note that the degree of any vertex is the sum of the entries in the corresponding row of the adjacency matrix. With this in mind, the problem is not so hard. Denote by

the adjacency matrix of

for ease of typing. Let

be an eigenvector of

with associated eigenvalue
, and let

be the largest entry in
. Note that by the definition of matrix multiplication and our condition of maximality
![\[(A\vec{x})_i=\sum_jA_{ij}x_j\leq\sum_jA_{ij}x_i=x_i\sum_jA_{ij}\leq\Delta x_i.\]](//latex.artofproblemsolving.com/e/0/5/e056c8c176b3eefc0cec784f90843cbfe041dd84.png)
But since

is an eigenvector we have
. Hence equating entries yields

for any eigenvalue
, from which the result follows.
(b)This problem is a bit harder (or at least I think it is - perhaps I'm overcomplicating things like I usually do...). The key is to note that since

is simple, all entries are either

or

and that the sum of the entries in the matrix

is
.
Note that from

we can take norms to get
, which implies
.
Now denote by
,
,
, 
the entries of
. I will show that
![\[\sum_{i=1}^n\left(\sum_jA_{ij}x_j\right)^2\leq 2q\sum_{j=1}^nx_j^2.\]](//latex.artofproblemsolving.com/6/5/c/65c4cb4d0e6998e5c58523f188593f1633faf98b.png)
Once this is proven, the LHS becomes

while the right becomes
, from which the conclusion is easy to reach.
To do this, we use Cauchy-Schwarz. Specifically, note that
![\[\sum_{j=1}^nA_{ij}^2\sum_{j=1}^nx_{ij}^2\geq\left(\sum_{j=1}^nA_{ij}x_j\right)^2.\]](//latex.artofproblemsolving.com/0/3/b/03b7115e1f3dccb7e59531468db915aed07ee26a.png)
Summing over all

yields
![\[\sum_{i=1}^n\left(\sum_{j=1}^nA_{ij}x_j\right)^2\leq\sum_{i=1}^n\left(\sum_{j=1}^nA_{ij}^2\sum_{j=1}^nx_j^2\right)\leq\left(\sum_{i,j}A_{ij}^2\right)\left(\sum_{j=1}^nx_j^2\right).\]](//latex.artofproblemsolving.com/b/c/5/bc5ee2b5ffbf393bb8cbfb01ded4045cee3b4655.png)
Finally, remark that since all entries in

are either

or
, 
simply equals the sum of the entries in
, which equals
. Thus we get the desired inequality, which proves the statement we set out to prove.
This post has been edited 3 times. Last edited by djmathman, Dec 28, 2015, 5:03 AM